The base and height in a figure are always what?

The Base And Height In A Figure Are Always What?

Answers

Answer 1

Answer:

Perpendicular

Step-by-step explanation:

By definition, the base and height of a figure must always intersect at a [tex]90^{\circ}[/tex] angle. Therefore, they are always perpendicular.


Related Questions

A plant costs p dollars and a bush costs b dollars. Ana buys 2 plants and 4 bushes for $42. Paola buys 7 plants and 9 bushes for $107. Write down a pair of simultaneous equations and solve them to find out the value of p and the value of b.

Answers

Answer:

2p + 4b = 42

7p + 9b = 107

p = $5

b = $8

Step-by-step explanation:

p is cost of plant, b is cost of bush

2p + 4b = 42

7p + 9b = 107

First we'll solve the first equation for p in terms of b:

2p + 4b = 42

subtract 4b from both sides

2p = -4b + 42

divide both sides by 2

p = -2b + 21

Now substitute that value in for p in the second equation, and solve for a numerical value of b:

7(-2b + 21) + 9b = 107

-14b + 147 + 9b = 107

combine like terms:

-5b = -40

b = 8

Use this to go back and solve for p:

p = -2b + 21

p = -2(8) + 21

p = -16 + 21 = 5

---------------------------

check the math:

2p + 4b = 42

2(5) + 4(8) = 10 + 32 = 42

2. What are the zeros of the function
f(x)=2x2 - 2x - 4?

Answers

Answer:

-1 or 2

Step-by-step explanation:

2x2-2x-4

x2-x-2

x2+x-2x-2

x(x+1)-2(x+1)

(x+1)(x-2)

x=-1 or 2

Una empresa quiere confecionar 7²⁰ chompas. Si el 1er día confeccionada los 3/10 del total. El 2do dia los 2/9 del total, ¿cuantas chompas le falta confeccionar? Con cada paso porfavor :(

Answers

Answer:

A la empresa le faltan confeccionar 344 chompas.

Step-by-step explanation:

Dado que una empresa quiere confeccionar 720 chompas, si el 1er día confeccionaron los 3/10 del total y el 2do día los 2/9 del total, para determinar cuántas chompas le falta confeccionar se debe realizar el siguiente cálculo:

Día 1:

720 x 3/10 = X

720 x 0.3 = X

216 = X

Día 2:

720 x 2/9 = X

720 x 0.22222 = X

160 = X

720 - 216 - 160 = X

344 = X

Por lo tanto, a la empresa le faltan confeccionar 344 chompas.

please help thank you

Answers


I don’t know it is really hard to learn

Answer:

50  ft²

Step-by-step explanation:

ΔABC ≅ ADC; therefore the area of ΔADC is 25 sq feet also

2(25) = 50 sq feet

What is an equation of the line that passes through the points (-5,6) and (7,6)?​

Answers

The y values are not changing so y = 6 describes the line

Which of the following is a geometric sequence?
O A. 2, 4, 8, 16, 32
O B. 3, 6, 9, 15, 24
O C. 6, 13, 20, 27, 34
O D. 2,5,7,12, 19

Answers

answer: A.

explanation: multiply 2 x 2= 4 then 4 x 2= 8 then 8 x 2= 16 then 16 x 2= 32 so basically your just multiplying every number by the same exact number which is 2

please find Area of triangle

Answers

Answer:

60

Step-by-step explanation:

Side A would end up being 15 and then to find the area you would use your formula 1/2 × base × height.

help mi plzz.........​

Answers

tan3A=tan(2A+A)

We know that , tan(x+y)=(tanx + tany)/(1 - (tanx)(tany))

tan(2A+A)=(tan2A+tanA)/(1 - (tan2A)(tanA))— (1)

We know that , tan2x=2tanx/(1 - tan^2x)

So, by substituting tan2A in (1),we get,

=[2tanA/(1 - tan^2A) + tanA]/1- (2tanA/(1 - tan^2A))(tanA)]

=[2tanA+tanA - tan^3A]/[1 - tan^2A - 2tan^2A]

=[3tanA - tan^3A]/[1-3tan^2A]

Therefore, tan3A= [3tanA - tan^3A]/[1-3tan^2A]

can someone please help for brainlest

Answers

No sorry I cannot help

(m ^ 2 * n ^ 3)/(n ^ - 2)

Answers

Answer:

m²n⁵

Step-by-step explanation:

m²n³ / n-²

The exponent changes sign when "flipped" from division to multiplication

m²n³ * n²

m²n⁵

Use the following situation to answer the questions that follow: A ball is thrown up in the air, and it's height (in feet) as a function of time (in seconds) can be written as h(t) = -16t2 + 32t + 6.

When is the ball at it's maximum height? Round to the nearest hundredth if needed.

Answers

Answer:

22 feet

Step-by-step explanation:

When graphing the equation, the peak or the maximum of the equation is reached the 1st second of the throw and it reaches 22 feet.

Edgar owns 234 shares of Cawh Consolidated Bank, which he bought for $21.38 apiece. Each share pays a yearly dividend of $3.15. Edgar also owns two par value $1,000 bonds from Cawh Consolidated Bank. The bonds had a market value of 105.166 when he bought them, and pay 8.3% interest yearly. Which aspect of Edgar’s investment in Cawh Consolidated Bank offers a greater percent yield, and how much greater is it? a. The stocks have a yield 6.43 percentage points greater than that of the bonds. b. The stocks have a yield 6.84 percentage points greater than that of the bonds. c. The bonds have a yield 1.05 percentage points greater than that of the stocks. d. The bonds have a yield 9.13 percentage points higher than that of the stocks.

Answers

Answer: B. The stocks have a yield 6.84 percentage points greater than that of the bonds.

Step-by-step explanation:

Firstly, the yield for stocks will be calculated as:

= return/ investment cost

= $3.15/$ 21.38

= 0.14733395

= 14.73%

The yield for bonds will be calculated as:

= Return/Investment cost

Return = 1,000 x 8.3% = 83

Investment cost = 1,000 x 105.166/100 = 1051.66‬

Yield = 83/1051.66

= 0.07892284

= 7.89%

Then, the difference between the yield will be:

= 14.73% - 7.89%

= 6.84%

Therefore, the stocks have a yield 6.84 percentage points greater than that of the bonds.

Answer:

b.The stocks have a yield 6.84 percentage points greater than that of the bonds.    Step-by-step explanation: got it right on edge 2022!

PlzzPlease help me bunnies <3
No monkey business plz. Or I will report

Defined the explicit section for the sqeucrce in the table
Thank you

Answers

Answer:

B. Linear decreasing

Step-by-step explanation:

Answer:

Linear decreasing;

Straight line = linear

Slope downwards (from left to right) = decreasing function

[tex]\sqrt \frac{256}{4\\}[/tex]

Answers

Answer:

8

Step-by-step explanation:

[tex]\sqrt{\frac{256}{4} }[/tex]

[tex]\sqrt{\frac{16*16}{2*2} }[/tex]

[tex]\sqrt{(\frac{16}{2})^2 }[/tex]

square and root gets cancel . so ,

[tex]\frac{16}{2}[/tex]

8

What are the roots of the equation 9x^2-42x+50=0 in simplest a+bi form?

Answers

Answer:

-7/3 +\- 1/3i

Step-by-step explanation:

If Arnold charges $522.00 for a 4 personal training sessions (of an hour each)
a) What is his hourly rate?
b) How much would he charge for a 3 hour session?

Answers

Answer:

Hourly rate = 130.50

3-hour session = 391.50

Step-by-step explanation:

Part 1

Step 1: Divide the given figures

522.00/4 = 130.50

Step 2: Multiply the result to the number being looked for

130.50 * 1 = 130.50

So, Arnold’s hourly rate is 130.50

Part 2

Step 1: Multiply the answer found in part one to 3

130.50 * 3 = 391.50

Therefore, Arnold’s would charge $391.50 in a 3-hour session

one month Laura rented 3 movies and 2 video games for a total of 22$. The next month she rented 5 movies and 6 video games for a total of $52. Find the rental cost for each movie and each video game.

Answers

Step-by-step explanation:

24$

There is 1 box of plain yogurt with 18 yogurt cups. There are 4 boxes of fruit yogurt each with c yogurt cups. Represent this situation with a picture or diagram.​

Answers

Hope this helps :D
(The number or letter written inside is the number of cups)

Cash in hand $79; Bank overdraft $27​

Answers

Answer:

Step-by-step explanation:

cash in bank = 79

overdraft = 27

left = 79 - 27

     = 52

SOMEONE ONE PLEASE HELP

Answers

Answer:

ummmmmmmmmmmmmmmmmmmmmmmmmmmmmmmmmmmmmmmmmmmmmmmmmmmmmmmmmmmmmmmm

Step-by-step explanation:

Find measure of angles B, E, D and C and the length of BC


Triangle ABC ~ Triangle DEF. Measure of angle A = 30 degrees and measure of angle F = 65 degrees. AB=20, DE = 35, EF= 28.

Answers

Answer:

< B = 180-30-65=85°

< E = 85°

< D = 30°

< C = 65°

BC = 20/35 × 28 = 16

Please help me!! I really need it!

Answers

Answer:

X= 7.5 Y=75

Step-by-step explanation:

If the equation y=-5x+12 is translated up five units, what would the new equation be?

Answers

Answer:

y=-5x+17

Step-by-step explanation:

Answer:

[see below]

Step-by-step explanation:

We would add '5' to the y-intercept (12) to make a translation up 5 units.

y = -5x + 12 + 5

y = -5x + 17

Hope this helps.

What is the solution to the system of equations ?
Y=2/3 x + 3
X= - 2

Answers

Answer:

x = -2, y = 5/3

Step-by-step explanation:

we have the value of x already (-2).

x=-2

Plug it into the first equation y = 2/3x+3

y = 2/3(-2)+3

y = -4/3+3

y = -4/3+9/3

y = 5/3

x = -2, y = 5/3

15.
Which of the quadratic functions has the widest graph?

A. 0.3x2

B. -4x2

C. -4/5x2

D. 1/3x2

Answers

If I’m correct, I think A. 0.3x2 is the correct answer.

Answer:

A. 0.3x2

Step-by-step explanation:

Graph the parabola using the direction, vertex, focus, and axis of symmetry.

Direction: Opens Up

Vertex:  

(

0

,

0

)

Focus:  

(

0

,

0.8

¯

3

)

Axis of Symmetry:  

x

=

0

Directrix:  

y

=

0.8

¯

3

x

y

2

1.2

1

0.3

0

0

1

0.3

2

1.2

Graph the parabola using the direction, vertex, focus, and axis of symmetry.

Direction: Opens Down

Vertex:  

(

0

,

0

)

Focus:  

(

0

,

1

16

)

Axis of Symmetry:  

x

=

0

Directrix:  

y

=

1

16

x

y

2

16

1

4

0

0

1

4

2

16

Graph the parabola using the direction, vertex, focus, and axis of symmetry.

Direction: Opens Down

Vertex:  

(

0

,

0

)

Focus:  

(

0

,

5

16

)

Axis of Symmetry:  

x

=

0

Directrix:  

y

=

5

16

x

y

2

16

5

1

4

5

0

0

1

4

5

2

16

5

Graph the parabola using the direction, vertex, focus, and axis of symmetry.

Direction: Opens Up

Vertex:  

(

0

,

0

)

Focus:  

(

0

,

3

4

)

Axis of Symmetry:  

x

=

0

Directrix:  

y

=

3

4

x

y

2

4

3

1

1

3

0

0

1

1

3

3

3

Factor completely 25x^4y^6 − 1

Answers

Answer:

(5x2y3+1)(5x2y3−1)

Step-by-step explanation:

In this case, since this equation has both sides as a perfect square, you can apply the difference of squares formula a^2 - b^2 = (a+b) (a-b)

Applying this formula and factoring it accordingly leaves us off with (5x2y3+1)(5x2y3−1).

x+y=80
5x+8y=535

what is x and what is y

Answers

Answer:

x+y=80 i.e. 5x+5y=400 multiplying both sides by 5

now 5x+8y=535

therefore 3y=535-400=135

therefore y=135/3=45

now x=80-y=80-45=35

therefore final answer x is 35 y is 45

Answer:

x=35, y=45

Step-by-step explanation:

x+y=80=>5x+5y=400 so (5x+8y)-(5x+5y)=535-400

<=>3y=135<=>y=45=>x=80-y=80-45=35

Determine the values of m and if the straight line y = mx + c passes through the point (3, 5) and has a gradient -4 .

Answers

Answer:

y = -4x + 17

Step-by-step explanation:

y = -4x + b

5 = -4(3) + b

5 = -12 + b

17 = b

If x + x = 9, then x =

Answers

X= 4.5 because 4.5+4.5=9

Answer: x = 4.5

Step-by-step explanation: First, do x + x, since they are like terms. That equals 2x. The equation is now 2x = 9. Divide both sides by 2. x = 4.5.

  x + x = 9

  2x = 9

÷2    ÷2

---------------------

 x = 4.5  

x + x = 9

4.5 + 4.5 = 9

I am stuck on this question.

Answers

Answer:

D. Yes; the graph passes the vertical line test.

Step-by-step explanation:

1. A and C are wrong since the number of x-values that a y-value have does not matter when it comes to functions.

2. When doing the vertical line test, it is done by drawing a vertical line (up to down) and then seeing if more than 1 point touches the line.

3. After drawing the vertical line you would see only 1 point touches the line which indicates that the graph passes the vertical line test.

4. Therefore, the answer is "D. Yes; the graph passes the vertical line test."

Other Questions
Megan uses cup of almonds to make 4 cups of trail mix. Using thissame proportion, how many cups of almonds would Megan need to make9 cups of trail mix? During a school event, Noah is selling pizza at the food stand for $1.25 per slice. How many slices of pizza would Noah need to sell to earn $120? Explain your reasoning. Adults engaging in NO leisure-time physical activity has DECREASED since 2008.A: TrueB: False how a forensic scientist may match DNA of evidence to a particular crime? Which of the following is most likely to form when hot magma rises up as tectonic plates move apart below the ocean1.mid-ocean ridge2.fault3.trenches4.subduction zone 4-9+8+7+9+10+11+21+33+25-3 minus -50-55+50 The table of values shown below represents a linear function. Which of these points could also be an ordered pair in the table, and why? Current information for the Healey Company follows : Beginning raw materials inventory Raw material purchases Ending raw materials inventory Beginning work in process inventory Ending work in process inventory Direct labor Total factory overhead $ 15,700 60,500 17,100 22,900 28,500 43,30030,500 All raw materials used were traceable to specific units of product . Healey Company's direct materials used for the year is : Para que sirve el reciclaje de componentes elctricos? Que realizas t para realizar los dispositivos informticos que ya no utilizas? Which number is GREATEST?0.270.0460.2970 0.3 Pleaseee Helpp. If the radius is 21 m and area of sector of the circle are 27.72 m2, calculate the angle at the centre of the circle. Thanes were a group of upperclass, free noblemen whom the king used for military service.TrueFalse PLEASEEEE ANSWER THIS IM BEING TIMED AND SHOW YOUR WORK ILL GIVE TOU BRAINLIST IF YOU DO!!!! Help pls!!!!!!!!!!!!!! the height of a tower is 15m more than tiwce the height of a building find the height of the building if tower is 255m tall EllusFind the range of the data.The ages of people eatingchocolate ice cream at a park.12, 2, 8, 5, 3, 2,5Range: [? ]Enter the number that belongs in the green box. Kathie buys a bottle of fruit juice for $2.50 and a few loaves of bread for $1.20 each.a. If she spent $8.50, how many loaves of bread did she buy?b. If she can spend AT MOST $20, how many loaves of bread can she buy? An older sibling has 3 beers at happy hour then drives home. This is an example of A: Drug Abuse,B: Drug Use, C: Intoxication, Drug Misuse, 1:Under what condition will the line px+py+r=0 mat be a normal to the circke x+y+2gx+2fy+c=02:prove that the two circles x+y+2ax+c=0 and x+y+2by+c=0 touch if [tex]\frac{1}{a}+\frac{1}{b}=\frac{1}{c}[/tex] PLS HELP! Two angles are complementary. Draw a diagram that illustrates the complementary angles. One angle measures 35 and the other measures (x + 15). What is the value of x?